A number when divided by 2, 3, 4, 5, 6 leaves a remainder of 1 but it is divided by 7 completely.












7












$begingroup$


I came across a question which is as follows:



Find out the smallest number which leaves remainder of 1 when divided by 2, 3, 4, 5, 6 but divided by 7 completely.



What I did is given below step wise.



Step 1- Find out the LCM of 2, 3, 4, 5, 6 which is 60.



Step 2- Add 1 to 60 which is 61.



Step 3- Multiple 61 by 7 repeatedly till it fulfills the condition that remainder should be 1.



Step 4- I got the answer 146461 which seems to correct.



So now my question is:



1) Is this answer correct? If yes how to verify that this is smallest number which fulfills above condition?



2) I think this is not the best way to do this question. So Can anyone give a better way to solve this problem?



Thanks in advance










share|cite|improve this question











$endgroup$








  • 1




    $begingroup$
    301?? You could also use a system of congruence equations. By the way, you don't want to multiply by 61...Think about why...
    $endgroup$
    – Eleven-Eleven
    Jan 24 '15 at 3:44












  • $begingroup$
    How did you reach at 301 by using congruence relation? Can you pls explain it bit further?
    $endgroup$
    – Deepak Uniyal
    Jan 24 '15 at 3:51






  • 3




    $begingroup$
    The answer is less than $420$.
    $endgroup$
    – André Nicolas
    Jan 24 '15 at 4:02










  • $begingroup$
    Well, first and foremost you were on the right track with LCM of 2,3,4,5,6. Adding 1 yields a remainder of 1. So numbers that leave a remainder of 1 would be 60k+1.
    $endgroup$
    – Eleven-Eleven
    Jan 24 '15 at 4:03






  • 1




    $begingroup$
    @Eleven-Eleven This method works, because eventually $7^{k}equiv 1pmod{60}$, so starting with $61$ and repeatedly multiply by $7$ works. It would work just as well to start with $1$ and repeated multiplying by $7$, of course, or you could apply the Chinese Remainder Theorem, as inmy answer. Repeated multiplying will not find the smallest answer, of course.
    $endgroup$
    – Thomas Andrews
    Jan 24 '15 at 4:17


















7












$begingroup$


I came across a question which is as follows:



Find out the smallest number which leaves remainder of 1 when divided by 2, 3, 4, 5, 6 but divided by 7 completely.



What I did is given below step wise.



Step 1- Find out the LCM of 2, 3, 4, 5, 6 which is 60.



Step 2- Add 1 to 60 which is 61.



Step 3- Multiple 61 by 7 repeatedly till it fulfills the condition that remainder should be 1.



Step 4- I got the answer 146461 which seems to correct.



So now my question is:



1) Is this answer correct? If yes how to verify that this is smallest number which fulfills above condition?



2) I think this is not the best way to do this question. So Can anyone give a better way to solve this problem?



Thanks in advance










share|cite|improve this question











$endgroup$








  • 1




    $begingroup$
    301?? You could also use a system of congruence equations. By the way, you don't want to multiply by 61...Think about why...
    $endgroup$
    – Eleven-Eleven
    Jan 24 '15 at 3:44












  • $begingroup$
    How did you reach at 301 by using congruence relation? Can you pls explain it bit further?
    $endgroup$
    – Deepak Uniyal
    Jan 24 '15 at 3:51






  • 3




    $begingroup$
    The answer is less than $420$.
    $endgroup$
    – André Nicolas
    Jan 24 '15 at 4:02










  • $begingroup$
    Well, first and foremost you were on the right track with LCM of 2,3,4,5,6. Adding 1 yields a remainder of 1. So numbers that leave a remainder of 1 would be 60k+1.
    $endgroup$
    – Eleven-Eleven
    Jan 24 '15 at 4:03






  • 1




    $begingroup$
    @Eleven-Eleven This method works, because eventually $7^{k}equiv 1pmod{60}$, so starting with $61$ and repeatedly multiply by $7$ works. It would work just as well to start with $1$ and repeated multiplying by $7$, of course, or you could apply the Chinese Remainder Theorem, as inmy answer. Repeated multiplying will not find the smallest answer, of course.
    $endgroup$
    – Thomas Andrews
    Jan 24 '15 at 4:17
















7












7








7


5



$begingroup$


I came across a question which is as follows:



Find out the smallest number which leaves remainder of 1 when divided by 2, 3, 4, 5, 6 but divided by 7 completely.



What I did is given below step wise.



Step 1- Find out the LCM of 2, 3, 4, 5, 6 which is 60.



Step 2- Add 1 to 60 which is 61.



Step 3- Multiple 61 by 7 repeatedly till it fulfills the condition that remainder should be 1.



Step 4- I got the answer 146461 which seems to correct.



So now my question is:



1) Is this answer correct? If yes how to verify that this is smallest number which fulfills above condition?



2) I think this is not the best way to do this question. So Can anyone give a better way to solve this problem?



Thanks in advance










share|cite|improve this question











$endgroup$




I came across a question which is as follows:



Find out the smallest number which leaves remainder of 1 when divided by 2, 3, 4, 5, 6 but divided by 7 completely.



What I did is given below step wise.



Step 1- Find out the LCM of 2, 3, 4, 5, 6 which is 60.



Step 2- Add 1 to 60 which is 61.



Step 3- Multiple 61 by 7 repeatedly till it fulfills the condition that remainder should be 1.



Step 4- I got the answer 146461 which seems to correct.



So now my question is:



1) Is this answer correct? If yes how to verify that this is smallest number which fulfills above condition?



2) I think this is not the best way to do this question. So Can anyone give a better way to solve this problem?



Thanks in advance







puzzle least-common-multiple






share|cite|improve this question















share|cite|improve this question













share|cite|improve this question




share|cite|improve this question








edited Jan 25 '15 at 4:55









ReliableMathBoy

582618




582618










asked Jan 24 '15 at 3:41









Deepak UniyalDeepak Uniyal

38115




38115








  • 1




    $begingroup$
    301?? You could also use a system of congruence equations. By the way, you don't want to multiply by 61...Think about why...
    $endgroup$
    – Eleven-Eleven
    Jan 24 '15 at 3:44












  • $begingroup$
    How did you reach at 301 by using congruence relation? Can you pls explain it bit further?
    $endgroup$
    – Deepak Uniyal
    Jan 24 '15 at 3:51






  • 3




    $begingroup$
    The answer is less than $420$.
    $endgroup$
    – André Nicolas
    Jan 24 '15 at 4:02










  • $begingroup$
    Well, first and foremost you were on the right track with LCM of 2,3,4,5,6. Adding 1 yields a remainder of 1. So numbers that leave a remainder of 1 would be 60k+1.
    $endgroup$
    – Eleven-Eleven
    Jan 24 '15 at 4:03






  • 1




    $begingroup$
    @Eleven-Eleven This method works, because eventually $7^{k}equiv 1pmod{60}$, so starting with $61$ and repeatedly multiply by $7$ works. It would work just as well to start with $1$ and repeated multiplying by $7$, of course, or you could apply the Chinese Remainder Theorem, as inmy answer. Repeated multiplying will not find the smallest answer, of course.
    $endgroup$
    – Thomas Andrews
    Jan 24 '15 at 4:17
















  • 1




    $begingroup$
    301?? You could also use a system of congruence equations. By the way, you don't want to multiply by 61...Think about why...
    $endgroup$
    – Eleven-Eleven
    Jan 24 '15 at 3:44












  • $begingroup$
    How did you reach at 301 by using congruence relation? Can you pls explain it bit further?
    $endgroup$
    – Deepak Uniyal
    Jan 24 '15 at 3:51






  • 3




    $begingroup$
    The answer is less than $420$.
    $endgroup$
    – André Nicolas
    Jan 24 '15 at 4:02










  • $begingroup$
    Well, first and foremost you were on the right track with LCM of 2,3,4,5,6. Adding 1 yields a remainder of 1. So numbers that leave a remainder of 1 would be 60k+1.
    $endgroup$
    – Eleven-Eleven
    Jan 24 '15 at 4:03






  • 1




    $begingroup$
    @Eleven-Eleven This method works, because eventually $7^{k}equiv 1pmod{60}$, so starting with $61$ and repeatedly multiply by $7$ works. It would work just as well to start with $1$ and repeated multiplying by $7$, of course, or you could apply the Chinese Remainder Theorem, as inmy answer. Repeated multiplying will not find the smallest answer, of course.
    $endgroup$
    – Thomas Andrews
    Jan 24 '15 at 4:17










1




1




$begingroup$
301?? You could also use a system of congruence equations. By the way, you don't want to multiply by 61...Think about why...
$endgroup$
– Eleven-Eleven
Jan 24 '15 at 3:44






$begingroup$
301?? You could also use a system of congruence equations. By the way, you don't want to multiply by 61...Think about why...
$endgroup$
– Eleven-Eleven
Jan 24 '15 at 3:44














$begingroup$
How did you reach at 301 by using congruence relation? Can you pls explain it bit further?
$endgroup$
– Deepak Uniyal
Jan 24 '15 at 3:51




$begingroup$
How did you reach at 301 by using congruence relation? Can you pls explain it bit further?
$endgroup$
– Deepak Uniyal
Jan 24 '15 at 3:51




3




3




$begingroup$
The answer is less than $420$.
$endgroup$
– André Nicolas
Jan 24 '15 at 4:02




$begingroup$
The answer is less than $420$.
$endgroup$
– André Nicolas
Jan 24 '15 at 4:02












$begingroup$
Well, first and foremost you were on the right track with LCM of 2,3,4,5,6. Adding 1 yields a remainder of 1. So numbers that leave a remainder of 1 would be 60k+1.
$endgroup$
– Eleven-Eleven
Jan 24 '15 at 4:03




$begingroup$
Well, first and foremost you were on the right track with LCM of 2,3,4,5,6. Adding 1 yields a remainder of 1. So numbers that leave a remainder of 1 would be 60k+1.
$endgroup$
– Eleven-Eleven
Jan 24 '15 at 4:03




1




1




$begingroup$
@Eleven-Eleven This method works, because eventually $7^{k}equiv 1pmod{60}$, so starting with $61$ and repeatedly multiply by $7$ works. It would work just as well to start with $1$ and repeated multiplying by $7$, of course, or you could apply the Chinese Remainder Theorem, as inmy answer. Repeated multiplying will not find the smallest answer, of course.
$endgroup$
– Thomas Andrews
Jan 24 '15 at 4:17






$begingroup$
@Eleven-Eleven This method works, because eventually $7^{k}equiv 1pmod{60}$, so starting with $61$ and repeatedly multiply by $7$ works. It would work just as well to start with $1$ and repeated multiplying by $7$, of course, or you could apply the Chinese Remainder Theorem, as inmy answer. Repeated multiplying will not find the smallest answer, of course.
$endgroup$
– Thomas Andrews
Jan 24 '15 at 4:17












6 Answers
6






active

oldest

votes


















14












$begingroup$

You definitely need $nequiv 1pmod {60}$ and $nequiv 0pmod 7$. So the trick is to apply the Chinese remainder theorem. Solve $60x+7y=1$ with $(x,y)=(2,-17)$.



Then $nequiv 1cdot 7cdot (-17)+0cdot 60cdot 2pmod{420}$, or $nequiv -119pmod{420}$. The smallest such positive number is $420-119=301$.






share|cite|improve this answer









$endgroup$





















    4












    $begingroup$

    Consider $60k+1,$ $$60(7n)+1=420n+1$$ $$60(7n+1)+1=420n+61$$ $$60(7n+2)+1=420n+121$$ $$60(7n+3)+1=420n+181$$ $$60(7n+4)+1=420n+241$$ $$60(7n+5)+1=420n+301$$ $$60(7n+6)+1=420n+361.$$ Now by divisibility test for 7, among $1, 61, 121, 181, 241, 301, 361$, only $301$ is divisible by $7.$



    Therefore any number of the form $420n+301$ satisfies the given requirements.






    share|cite|improve this answer











    $endgroup$





















      3












      $begingroup$

      Here's another way to tackle it. You already figured out that you're looking for $60k+1$. When you multiply $60$ by $k$ you want a predecessor to a multiplication of $7$.



      $60 equiv 4 pmod 7$ and $5 times 4 = 20$ which is a predecessor to a multiplication of $7$.



      So $k=5$.






      share|cite|improve this answer











      $endgroup$









      • 3




        $begingroup$
        The OP didn't "already figure out that [he's] looking for $60k+1$". He thought he was looking for $61cdot 7^k$.
        $endgroup$
        – Henning Makholm
        Jan 24 '15 at 12:58



















      2












      $begingroup$

      Since no-one has mentioned it in their answers so far, your step 3 is wrong, or at least ill-advised, in that you are coming across a lot of answers that do not meet your carefully-set-up satisfaction of the first five conditions. For example, $61times 7 = 427$ does not meet the desired remainders for $4$ or $5$.



      The problem is that you have abandoned the safety of $nequiv 1 bmod 60$. The way to retain this in a simple search is to add 60 repeatedly looking for divisibility by 7.



      We can do a bit better than that, though. We can translate into smaller numbers to make life easier for ourselves. $60 equiv 4 bmod 7$ (and of course $61 equiv 5 bmod 7$) so we can ask: how many 4s do we need to add to 5 before the answer is divisible by 7?



      Again we can slog through the possibilities but the multiples of 7 are easier to cope with. We add two 4s (13 - and maybe see that we've reached $6 bmod 7$) and add another two 4s to reach $21 equiv 7 equiv 0 bmod 7$ - adding four 4s altogether. So back on our Actual Problem, we need to add four 60s - $4times 60=240$ to our original $61$ so $ 240+61=301$ for the smallest positive solution.



      Note that $60$ is your interval between satisfying those first 5 conditions, but we could also have started our search at $1$ rathe than $61$, with (of course) the same eventual result.






      share|cite|improve this answer









      $endgroup$





















        2












        $begingroup$

        Below are a few different approaches (besides the standard Extended Euclidean Algorithm).



        ${rm mod} 60!: xequiv 1equiv 7n!iff! nequivdfrac{1}7equiv dfrac{-59}7equiv dfrac{-119}7equiv -17,$



        therefore $smash[t]{,x = 7(overbrace{-17!+!60k}^{large n}) = 420k-119}$



        Alternatively $, $ mod $,60!: color{#c00}{7^4equiv 1},$ (by true mod $3,4,5),$ so $smash[b]{,color{#c00}{7^{-1}equiv 7^3}equiv 7(underbrace{-11}_{Large 7^2})equiv -17}$



        Alternatively $ $ we can employ $ $ Inverse Reciprocity



        $qquad dfrac{1}7 {rm mod} 60 equiv dfrac{1-60overbrace{left(color{#c00}{dfrac{1}{60}} {rm mod} 7right)}^{Large color{#0a0}{equiv, 2}}}7,equiv, dfrac{-119}7 ,equiv, -17 $



        $text{where we've used} {rm mod} 7!:, color{#c00}{dfrac{1}{60}}equiv dfrac{8}4color{#0a0}{equiv 2}, $ (or recurse on $,dfrac{1}{60}bmod 7 ,equiv, dfrac{1}4 bmod 7,)$



        Remark $ $ In the first method we found a numerator $,-119equiv 1pmod{60}$ that's also divisible by $7$ by brute force, i.e. we tested $,1-60k,$ for $,k=1,2ldots$ But now we see that the solution $,color{#0a0}{ kequiv 2},$ is simply the reciprocal inverse $, k, =, 1/60,bmod, 7.,$



        Beware $ $ Modular fraction arithmetic is well-defined only for fractions with denominator coprime to the modulus. See here for further discussion.






        share|cite|improve this answer











        $endgroup$





















          0












          $begingroup$

          From $xequiv 1 pmod{60}$ we can proceed by adding the modulus until we find something congruent to $0pmod{7}$:



          $pmod{60}:xequiv 1 equiv 61 equiv 121 equiv 181 equiv 241 equiv 301$.



          Since $301equiv 0pmod{7}$, it gives the minimal solution.






          share|cite|improve this answer









          $endgroup$












            protected by Community Jun 14 '15 at 4:02



            Thank you for your interest in this question.
            Because it has attracted low-quality or spam answers that had to be removed, posting an answer now requires 10 reputation on this site (the association bonus does not count).



            Would you like to answer one of these unanswered questions instead?














            6 Answers
            6






            active

            oldest

            votes








            6 Answers
            6






            active

            oldest

            votes









            active

            oldest

            votes






            active

            oldest

            votes









            14












            $begingroup$

            You definitely need $nequiv 1pmod {60}$ and $nequiv 0pmod 7$. So the trick is to apply the Chinese remainder theorem. Solve $60x+7y=1$ with $(x,y)=(2,-17)$.



            Then $nequiv 1cdot 7cdot (-17)+0cdot 60cdot 2pmod{420}$, or $nequiv -119pmod{420}$. The smallest such positive number is $420-119=301$.






            share|cite|improve this answer









            $endgroup$


















              14












              $begingroup$

              You definitely need $nequiv 1pmod {60}$ and $nequiv 0pmod 7$. So the trick is to apply the Chinese remainder theorem. Solve $60x+7y=1$ with $(x,y)=(2,-17)$.



              Then $nequiv 1cdot 7cdot (-17)+0cdot 60cdot 2pmod{420}$, or $nequiv -119pmod{420}$. The smallest such positive number is $420-119=301$.






              share|cite|improve this answer









              $endgroup$
















                14












                14








                14





                $begingroup$

                You definitely need $nequiv 1pmod {60}$ and $nequiv 0pmod 7$. So the trick is to apply the Chinese remainder theorem. Solve $60x+7y=1$ with $(x,y)=(2,-17)$.



                Then $nequiv 1cdot 7cdot (-17)+0cdot 60cdot 2pmod{420}$, or $nequiv -119pmod{420}$. The smallest such positive number is $420-119=301$.






                share|cite|improve this answer









                $endgroup$



                You definitely need $nequiv 1pmod {60}$ and $nequiv 0pmod 7$. So the trick is to apply the Chinese remainder theorem. Solve $60x+7y=1$ with $(x,y)=(2,-17)$.



                Then $nequiv 1cdot 7cdot (-17)+0cdot 60cdot 2pmod{420}$, or $nequiv -119pmod{420}$. The smallest such positive number is $420-119=301$.







                share|cite|improve this answer












                share|cite|improve this answer



                share|cite|improve this answer










                answered Jan 24 '15 at 4:09









                Thomas AndrewsThomas Andrews

                130k12147298




                130k12147298























                    4












                    $begingroup$

                    Consider $60k+1,$ $$60(7n)+1=420n+1$$ $$60(7n+1)+1=420n+61$$ $$60(7n+2)+1=420n+121$$ $$60(7n+3)+1=420n+181$$ $$60(7n+4)+1=420n+241$$ $$60(7n+5)+1=420n+301$$ $$60(7n+6)+1=420n+361.$$ Now by divisibility test for 7, among $1, 61, 121, 181, 241, 301, 361$, only $301$ is divisible by $7.$



                    Therefore any number of the form $420n+301$ satisfies the given requirements.






                    share|cite|improve this answer











                    $endgroup$


















                      4












                      $begingroup$

                      Consider $60k+1,$ $$60(7n)+1=420n+1$$ $$60(7n+1)+1=420n+61$$ $$60(7n+2)+1=420n+121$$ $$60(7n+3)+1=420n+181$$ $$60(7n+4)+1=420n+241$$ $$60(7n+5)+1=420n+301$$ $$60(7n+6)+1=420n+361.$$ Now by divisibility test for 7, among $1, 61, 121, 181, 241, 301, 361$, only $301$ is divisible by $7.$



                      Therefore any number of the form $420n+301$ satisfies the given requirements.






                      share|cite|improve this answer











                      $endgroup$
















                        4












                        4








                        4





                        $begingroup$

                        Consider $60k+1,$ $$60(7n)+1=420n+1$$ $$60(7n+1)+1=420n+61$$ $$60(7n+2)+1=420n+121$$ $$60(7n+3)+1=420n+181$$ $$60(7n+4)+1=420n+241$$ $$60(7n+5)+1=420n+301$$ $$60(7n+6)+1=420n+361.$$ Now by divisibility test for 7, among $1, 61, 121, 181, 241, 301, 361$, only $301$ is divisible by $7.$



                        Therefore any number of the form $420n+301$ satisfies the given requirements.






                        share|cite|improve this answer











                        $endgroup$



                        Consider $60k+1,$ $$60(7n)+1=420n+1$$ $$60(7n+1)+1=420n+61$$ $$60(7n+2)+1=420n+121$$ $$60(7n+3)+1=420n+181$$ $$60(7n+4)+1=420n+241$$ $$60(7n+5)+1=420n+301$$ $$60(7n+6)+1=420n+361.$$ Now by divisibility test for 7, among $1, 61, 121, 181, 241, 301, 361$, only $301$ is divisible by $7.$



                        Therefore any number of the form $420n+301$ satisfies the given requirements.







                        share|cite|improve this answer














                        share|cite|improve this answer



                        share|cite|improve this answer








                        edited Jan 25 '15 at 4:56









                        ReliableMathBoy

                        582618




                        582618










                        answered Jan 24 '15 at 4:36









                        BumblebeeBumblebee

                        9,75612551




                        9,75612551























                            3












                            $begingroup$

                            Here's another way to tackle it. You already figured out that you're looking for $60k+1$. When you multiply $60$ by $k$ you want a predecessor to a multiplication of $7$.



                            $60 equiv 4 pmod 7$ and $5 times 4 = 20$ which is a predecessor to a multiplication of $7$.



                            So $k=5$.






                            share|cite|improve this answer











                            $endgroup$









                            • 3




                              $begingroup$
                              The OP didn't "already figure out that [he's] looking for $60k+1$". He thought he was looking for $61cdot 7^k$.
                              $endgroup$
                              – Henning Makholm
                              Jan 24 '15 at 12:58
















                            3












                            $begingroup$

                            Here's another way to tackle it. You already figured out that you're looking for $60k+1$. When you multiply $60$ by $k$ you want a predecessor to a multiplication of $7$.



                            $60 equiv 4 pmod 7$ and $5 times 4 = 20$ which is a predecessor to a multiplication of $7$.



                            So $k=5$.






                            share|cite|improve this answer











                            $endgroup$









                            • 3




                              $begingroup$
                              The OP didn't "already figure out that [he's] looking for $60k+1$". He thought he was looking for $61cdot 7^k$.
                              $endgroup$
                              – Henning Makholm
                              Jan 24 '15 at 12:58














                            3












                            3








                            3





                            $begingroup$

                            Here's another way to tackle it. You already figured out that you're looking for $60k+1$. When you multiply $60$ by $k$ you want a predecessor to a multiplication of $7$.



                            $60 equiv 4 pmod 7$ and $5 times 4 = 20$ which is a predecessor to a multiplication of $7$.



                            So $k=5$.






                            share|cite|improve this answer











                            $endgroup$



                            Here's another way to tackle it. You already figured out that you're looking for $60k+1$. When you multiply $60$ by $k$ you want a predecessor to a multiplication of $7$.



                            $60 equiv 4 pmod 7$ and $5 times 4 = 20$ which is a predecessor to a multiplication of $7$.



                            So $k=5$.







                            share|cite|improve this answer














                            share|cite|improve this answer



                            share|cite|improve this answer








                            edited Jan 24 '15 at 4:38

























                            answered Jan 24 '15 at 4:25









                            benjibenji

                            2,2681614




                            2,2681614








                            • 3




                              $begingroup$
                              The OP didn't "already figure out that [he's] looking for $60k+1$". He thought he was looking for $61cdot 7^k$.
                              $endgroup$
                              – Henning Makholm
                              Jan 24 '15 at 12:58














                            • 3




                              $begingroup$
                              The OP didn't "already figure out that [he's] looking for $60k+1$". He thought he was looking for $61cdot 7^k$.
                              $endgroup$
                              – Henning Makholm
                              Jan 24 '15 at 12:58








                            3




                            3




                            $begingroup$
                            The OP didn't "already figure out that [he's] looking for $60k+1$". He thought he was looking for $61cdot 7^k$.
                            $endgroup$
                            – Henning Makholm
                            Jan 24 '15 at 12:58




                            $begingroup$
                            The OP didn't "already figure out that [he's] looking for $60k+1$". He thought he was looking for $61cdot 7^k$.
                            $endgroup$
                            – Henning Makholm
                            Jan 24 '15 at 12:58











                            2












                            $begingroup$

                            Since no-one has mentioned it in their answers so far, your step 3 is wrong, or at least ill-advised, in that you are coming across a lot of answers that do not meet your carefully-set-up satisfaction of the first five conditions. For example, $61times 7 = 427$ does not meet the desired remainders for $4$ or $5$.



                            The problem is that you have abandoned the safety of $nequiv 1 bmod 60$. The way to retain this in a simple search is to add 60 repeatedly looking for divisibility by 7.



                            We can do a bit better than that, though. We can translate into smaller numbers to make life easier for ourselves. $60 equiv 4 bmod 7$ (and of course $61 equiv 5 bmod 7$) so we can ask: how many 4s do we need to add to 5 before the answer is divisible by 7?



                            Again we can slog through the possibilities but the multiples of 7 are easier to cope with. We add two 4s (13 - and maybe see that we've reached $6 bmod 7$) and add another two 4s to reach $21 equiv 7 equiv 0 bmod 7$ - adding four 4s altogether. So back on our Actual Problem, we need to add four 60s - $4times 60=240$ to our original $61$ so $ 240+61=301$ for the smallest positive solution.



                            Note that $60$ is your interval between satisfying those first 5 conditions, but we could also have started our search at $1$ rathe than $61$, with (of course) the same eventual result.






                            share|cite|improve this answer









                            $endgroup$


















                              2












                              $begingroup$

                              Since no-one has mentioned it in their answers so far, your step 3 is wrong, or at least ill-advised, in that you are coming across a lot of answers that do not meet your carefully-set-up satisfaction of the first five conditions. For example, $61times 7 = 427$ does not meet the desired remainders for $4$ or $5$.



                              The problem is that you have abandoned the safety of $nequiv 1 bmod 60$. The way to retain this in a simple search is to add 60 repeatedly looking for divisibility by 7.



                              We can do a bit better than that, though. We can translate into smaller numbers to make life easier for ourselves. $60 equiv 4 bmod 7$ (and of course $61 equiv 5 bmod 7$) so we can ask: how many 4s do we need to add to 5 before the answer is divisible by 7?



                              Again we can slog through the possibilities but the multiples of 7 are easier to cope with. We add two 4s (13 - and maybe see that we've reached $6 bmod 7$) and add another two 4s to reach $21 equiv 7 equiv 0 bmod 7$ - adding four 4s altogether. So back on our Actual Problem, we need to add four 60s - $4times 60=240$ to our original $61$ so $ 240+61=301$ for the smallest positive solution.



                              Note that $60$ is your interval between satisfying those first 5 conditions, but we could also have started our search at $1$ rathe than $61$, with (of course) the same eventual result.






                              share|cite|improve this answer









                              $endgroup$
















                                2












                                2








                                2





                                $begingroup$

                                Since no-one has mentioned it in their answers so far, your step 3 is wrong, or at least ill-advised, in that you are coming across a lot of answers that do not meet your carefully-set-up satisfaction of the first five conditions. For example, $61times 7 = 427$ does not meet the desired remainders for $4$ or $5$.



                                The problem is that you have abandoned the safety of $nequiv 1 bmod 60$. The way to retain this in a simple search is to add 60 repeatedly looking for divisibility by 7.



                                We can do a bit better than that, though. We can translate into smaller numbers to make life easier for ourselves. $60 equiv 4 bmod 7$ (and of course $61 equiv 5 bmod 7$) so we can ask: how many 4s do we need to add to 5 before the answer is divisible by 7?



                                Again we can slog through the possibilities but the multiples of 7 are easier to cope with. We add two 4s (13 - and maybe see that we've reached $6 bmod 7$) and add another two 4s to reach $21 equiv 7 equiv 0 bmod 7$ - adding four 4s altogether. So back on our Actual Problem, we need to add four 60s - $4times 60=240$ to our original $61$ so $ 240+61=301$ for the smallest positive solution.



                                Note that $60$ is your interval between satisfying those first 5 conditions, but we could also have started our search at $1$ rathe than $61$, with (of course) the same eventual result.






                                share|cite|improve this answer









                                $endgroup$



                                Since no-one has mentioned it in their answers so far, your step 3 is wrong, or at least ill-advised, in that you are coming across a lot of answers that do not meet your carefully-set-up satisfaction of the first five conditions. For example, $61times 7 = 427$ does not meet the desired remainders for $4$ or $5$.



                                The problem is that you have abandoned the safety of $nequiv 1 bmod 60$. The way to retain this in a simple search is to add 60 repeatedly looking for divisibility by 7.



                                We can do a bit better than that, though. We can translate into smaller numbers to make life easier for ourselves. $60 equiv 4 bmod 7$ (and of course $61 equiv 5 bmod 7$) so we can ask: how many 4s do we need to add to 5 before the answer is divisible by 7?



                                Again we can slog through the possibilities but the multiples of 7 are easier to cope with. We add two 4s (13 - and maybe see that we've reached $6 bmod 7$) and add another two 4s to reach $21 equiv 7 equiv 0 bmod 7$ - adding four 4s altogether. So back on our Actual Problem, we need to add four 60s - $4times 60=240$ to our original $61$ so $ 240+61=301$ for the smallest positive solution.



                                Note that $60$ is your interval between satisfying those first 5 conditions, but we could also have started our search at $1$ rathe than $61$, with (of course) the same eventual result.







                                share|cite|improve this answer












                                share|cite|improve this answer



                                share|cite|improve this answer










                                answered Jan 27 '15 at 23:11









                                JoffanJoffan

                                32.5k43269




                                32.5k43269























                                    2












                                    $begingroup$

                                    Below are a few different approaches (besides the standard Extended Euclidean Algorithm).



                                    ${rm mod} 60!: xequiv 1equiv 7n!iff! nequivdfrac{1}7equiv dfrac{-59}7equiv dfrac{-119}7equiv -17,$



                                    therefore $smash[t]{,x = 7(overbrace{-17!+!60k}^{large n}) = 420k-119}$



                                    Alternatively $, $ mod $,60!: color{#c00}{7^4equiv 1},$ (by true mod $3,4,5),$ so $smash[b]{,color{#c00}{7^{-1}equiv 7^3}equiv 7(underbrace{-11}_{Large 7^2})equiv -17}$



                                    Alternatively $ $ we can employ $ $ Inverse Reciprocity



                                    $qquad dfrac{1}7 {rm mod} 60 equiv dfrac{1-60overbrace{left(color{#c00}{dfrac{1}{60}} {rm mod} 7right)}^{Large color{#0a0}{equiv, 2}}}7,equiv, dfrac{-119}7 ,equiv, -17 $



                                    $text{where we've used} {rm mod} 7!:, color{#c00}{dfrac{1}{60}}equiv dfrac{8}4color{#0a0}{equiv 2}, $ (or recurse on $,dfrac{1}{60}bmod 7 ,equiv, dfrac{1}4 bmod 7,)$



                                    Remark $ $ In the first method we found a numerator $,-119equiv 1pmod{60}$ that's also divisible by $7$ by brute force, i.e. we tested $,1-60k,$ for $,k=1,2ldots$ But now we see that the solution $,color{#0a0}{ kequiv 2},$ is simply the reciprocal inverse $, k, =, 1/60,bmod, 7.,$



                                    Beware $ $ Modular fraction arithmetic is well-defined only for fractions with denominator coprime to the modulus. See here for further discussion.






                                    share|cite|improve this answer











                                    $endgroup$


















                                      2












                                      $begingroup$

                                      Below are a few different approaches (besides the standard Extended Euclidean Algorithm).



                                      ${rm mod} 60!: xequiv 1equiv 7n!iff! nequivdfrac{1}7equiv dfrac{-59}7equiv dfrac{-119}7equiv -17,$



                                      therefore $smash[t]{,x = 7(overbrace{-17!+!60k}^{large n}) = 420k-119}$



                                      Alternatively $, $ mod $,60!: color{#c00}{7^4equiv 1},$ (by true mod $3,4,5),$ so $smash[b]{,color{#c00}{7^{-1}equiv 7^3}equiv 7(underbrace{-11}_{Large 7^2})equiv -17}$



                                      Alternatively $ $ we can employ $ $ Inverse Reciprocity



                                      $qquad dfrac{1}7 {rm mod} 60 equiv dfrac{1-60overbrace{left(color{#c00}{dfrac{1}{60}} {rm mod} 7right)}^{Large color{#0a0}{equiv, 2}}}7,equiv, dfrac{-119}7 ,equiv, -17 $



                                      $text{where we've used} {rm mod} 7!:, color{#c00}{dfrac{1}{60}}equiv dfrac{8}4color{#0a0}{equiv 2}, $ (or recurse on $,dfrac{1}{60}bmod 7 ,equiv, dfrac{1}4 bmod 7,)$



                                      Remark $ $ In the first method we found a numerator $,-119equiv 1pmod{60}$ that's also divisible by $7$ by brute force, i.e. we tested $,1-60k,$ for $,k=1,2ldots$ But now we see that the solution $,color{#0a0}{ kequiv 2},$ is simply the reciprocal inverse $, k, =, 1/60,bmod, 7.,$



                                      Beware $ $ Modular fraction arithmetic is well-defined only for fractions with denominator coprime to the modulus. See here for further discussion.






                                      share|cite|improve this answer











                                      $endgroup$
















                                        2












                                        2








                                        2





                                        $begingroup$

                                        Below are a few different approaches (besides the standard Extended Euclidean Algorithm).



                                        ${rm mod} 60!: xequiv 1equiv 7n!iff! nequivdfrac{1}7equiv dfrac{-59}7equiv dfrac{-119}7equiv -17,$



                                        therefore $smash[t]{,x = 7(overbrace{-17!+!60k}^{large n}) = 420k-119}$



                                        Alternatively $, $ mod $,60!: color{#c00}{7^4equiv 1},$ (by true mod $3,4,5),$ so $smash[b]{,color{#c00}{7^{-1}equiv 7^3}equiv 7(underbrace{-11}_{Large 7^2})equiv -17}$



                                        Alternatively $ $ we can employ $ $ Inverse Reciprocity



                                        $qquad dfrac{1}7 {rm mod} 60 equiv dfrac{1-60overbrace{left(color{#c00}{dfrac{1}{60}} {rm mod} 7right)}^{Large color{#0a0}{equiv, 2}}}7,equiv, dfrac{-119}7 ,equiv, -17 $



                                        $text{where we've used} {rm mod} 7!:, color{#c00}{dfrac{1}{60}}equiv dfrac{8}4color{#0a0}{equiv 2}, $ (or recurse on $,dfrac{1}{60}bmod 7 ,equiv, dfrac{1}4 bmod 7,)$



                                        Remark $ $ In the first method we found a numerator $,-119equiv 1pmod{60}$ that's also divisible by $7$ by brute force, i.e. we tested $,1-60k,$ for $,k=1,2ldots$ But now we see that the solution $,color{#0a0}{ kequiv 2},$ is simply the reciprocal inverse $, k, =, 1/60,bmod, 7.,$



                                        Beware $ $ Modular fraction arithmetic is well-defined only for fractions with denominator coprime to the modulus. See here for further discussion.






                                        share|cite|improve this answer











                                        $endgroup$



                                        Below are a few different approaches (besides the standard Extended Euclidean Algorithm).



                                        ${rm mod} 60!: xequiv 1equiv 7n!iff! nequivdfrac{1}7equiv dfrac{-59}7equiv dfrac{-119}7equiv -17,$



                                        therefore $smash[t]{,x = 7(overbrace{-17!+!60k}^{large n}) = 420k-119}$



                                        Alternatively $, $ mod $,60!: color{#c00}{7^4equiv 1},$ (by true mod $3,4,5),$ so $smash[b]{,color{#c00}{7^{-1}equiv 7^3}equiv 7(underbrace{-11}_{Large 7^2})equiv -17}$



                                        Alternatively $ $ we can employ $ $ Inverse Reciprocity



                                        $qquad dfrac{1}7 {rm mod} 60 equiv dfrac{1-60overbrace{left(color{#c00}{dfrac{1}{60}} {rm mod} 7right)}^{Large color{#0a0}{equiv, 2}}}7,equiv, dfrac{-119}7 ,equiv, -17 $



                                        $text{where we've used} {rm mod} 7!:, color{#c00}{dfrac{1}{60}}equiv dfrac{8}4color{#0a0}{equiv 2}, $ (or recurse on $,dfrac{1}{60}bmod 7 ,equiv, dfrac{1}4 bmod 7,)$



                                        Remark $ $ In the first method we found a numerator $,-119equiv 1pmod{60}$ that's also divisible by $7$ by brute force, i.e. we tested $,1-60k,$ for $,k=1,2ldots$ But now we see that the solution $,color{#0a0}{ kequiv 2},$ is simply the reciprocal inverse $, k, =, 1/60,bmod, 7.,$



                                        Beware $ $ Modular fraction arithmetic is well-defined only for fractions with denominator coprime to the modulus. See here for further discussion.







                                        share|cite|improve this answer














                                        share|cite|improve this answer



                                        share|cite|improve this answer








                                        edited Dec 17 '18 at 15:14

























                                        answered Jan 24 '15 at 4:49









                                        Bill DubuqueBill Dubuque

                                        212k29195652




                                        212k29195652























                                            0












                                            $begingroup$

                                            From $xequiv 1 pmod{60}$ we can proceed by adding the modulus until we find something congruent to $0pmod{7}$:



                                            $pmod{60}:xequiv 1 equiv 61 equiv 121 equiv 181 equiv 241 equiv 301$.



                                            Since $301equiv 0pmod{7}$, it gives the minimal solution.






                                            share|cite|improve this answer









                                            $endgroup$


















                                              0












                                              $begingroup$

                                              From $xequiv 1 pmod{60}$ we can proceed by adding the modulus until we find something congruent to $0pmod{7}$:



                                              $pmod{60}:xequiv 1 equiv 61 equiv 121 equiv 181 equiv 241 equiv 301$.



                                              Since $301equiv 0pmod{7}$, it gives the minimal solution.






                                              share|cite|improve this answer









                                              $endgroup$
















                                                0












                                                0








                                                0





                                                $begingroup$

                                                From $xequiv 1 pmod{60}$ we can proceed by adding the modulus until we find something congruent to $0pmod{7}$:



                                                $pmod{60}:xequiv 1 equiv 61 equiv 121 equiv 181 equiv 241 equiv 301$.



                                                Since $301equiv 0pmod{7}$, it gives the minimal solution.






                                                share|cite|improve this answer









                                                $endgroup$



                                                From $xequiv 1 pmod{60}$ we can proceed by adding the modulus until we find something congruent to $0pmod{7}$:



                                                $pmod{60}:xequiv 1 equiv 61 equiv 121 equiv 181 equiv 241 equiv 301$.



                                                Since $301equiv 0pmod{7}$, it gives the minimal solution.







                                                share|cite|improve this answer












                                                share|cite|improve this answer



                                                share|cite|improve this answer










                                                answered Jan 24 '15 at 4:26









                                                paw88789paw88789

                                                29.4k12349




                                                29.4k12349

















                                                    protected by Community Jun 14 '15 at 4:02



                                                    Thank you for your interest in this question.
                                                    Because it has attracted low-quality or spam answers that had to be removed, posting an answer now requires 10 reputation on this site (the association bonus does not count).



                                                    Would you like to answer one of these unanswered questions instead?



                                                    Popular posts from this blog

                                                    Le Mesnil-Réaume

                                                    Ida-Boy-Ed-Garten

                                                    web3.py web3.isConnected() returns false always